LSAT and Law School Admissions Forum

Get expert LSAT preparation and law school admissions advice from PowerScore Test Preparation.

 nicholaspavic
PowerScore Staff
  • PowerScore Staff
  • Posts: 271
  • Joined: Jun 12, 2017
|
#41271
Hi kelia!

Welcome to the forum. :-D The reason why Answer Option (E) is incorrect is that the stimulus is limited to "short term" impacts on rental units. We know nothing from the stimulus about long term raising of rents vis a vis rental unit shortage. The mention of a "disadvantage" here is creating somewhat of a shell game answer, but it is so undefined that it is not a reasonable inference from the stimulus. Thanks for the great question and let us know us know if this helps!
 deck1134
  • Posts: 160
  • Joined: Jun 11, 2018
|
#49505
(A) is incorrect because it is speculative, right? We know that they struggle to raise rents when rent controls are in effect. But we do not know that it is impossible.

Is that okay logic?

Thanks
 Matt_JB
  • Posts: 7
  • Joined: Sep 08, 2018
|
#58027
This is an annoyance to me. Answer D states “In many municipalities there is now, or eventually will be, a shortage of rental units.”

I understand the passage makes a claim towards this but the passage also implies something different and we are told to be thorough with these questions.

It says, “In many municipalities, SPECIFICALLY...”

In my mind when it says specifically, that effectively implies less than the many in which it started to say. This is why I eliminated D.

Can someone tell me why my logic is off here?
 Ben DiFabbio
PowerScore Staff
  • PowerScore Staff
  • Posts: 39
  • Joined: Aug 02, 2018
|
#58063
Matt_JB wrote:This is an annoyance to me. Answer D states “In many municipalities there is now, or eventually will be, a shortage of rental units.”

I understand the passage makes a claim towards this but the passage also implies something different and we are told to be thorough with these questions.

It says, “In many municipalities, SPECIFICALLY...”

In my mind when it says specifically, that effectively implies less than the many in which it started to say. This is why I eliminated D.

Can someone tell me why my logic is off here?
Hi Matt,

The "specifically [...]" clause in that sentence serves to define the group of "many municipalities" to which the sentence refers. From this clause -- "In many municipalities, specifically in all those where tenants of rent-control units have a secure hold on political power and can get rent-control ordinances enacted or repealed" -- we can infer that rent control units exist in many municipalities. The stimulus also states that rent control will bring about a shortage of units. Hence, answer choice D is supported.

"Many" is a numerically undefined term. "Specifically" does not imply "less than many," since "less than many" is similarly undefined. Without any corresponding numerical value, we just have to take the stimulus at its word that "many" means "many."

Hope that helps!
User avatar
 confused_pringle
  • Posts: 2
  • Joined: Aug 12, 2022
|
#96787
I'm having a hard time understanding why D is correct. Since the stimulus says "Unless they are used as strictly temporary measures..." doesn't this imply that the negative effects only result when rent-control ordinances are NOT used strictly as temporary measures? So couldn't it be the case that the people with power in the municipalities use rent-control strictly as temporary measures and therefore may not face the shortage of rental units?

Thank you for the help!
User avatar
 katehos
PowerScore Staff
  • PowerScore Staff
  • Posts: 184
  • Joined: Mar 31, 2022
|
#96790
Hi confused_pringle!

The first sentence of the stimulus effectively establishes that when rent-control is not temporary, rent-control has negative effects. However, we learn that alongside these negative effects of non-temporary rent-control comes an advantage: smaller rent increases. So, we don't really know if temporary rent-control measures would have negative effects or not. Additionally, we don't want to make any assumptions about what people will/won't do. Remember, since this is a Must Be True question, we must stick to the facts contained within the stimulus!

What we do know, on the other hand, is that renters in many municipalities WANT the short-term gain associated with non-temporary measures. This means it Must Be True that many municipalities do or will have a non-temporary rent control, which means they do or will have a shortage of rental units and smaller renter increases. This is exactly what (D) says so it is our correct answer!

I hope this helps :)
Kate
User avatar
 ericsilvagomez
  • Posts: 49
  • Joined: Oct 16, 2023
|
#104370
Hi,

Is answer choice D correct because it repeats the second sentence? As for the other answer choices, I did not pick them because I do not think they have much support.
 Robert Carroll
PowerScore Staff
  • PowerScore Staff
  • Posts: 1787
  • Joined: Dec 06, 2013
|
#104419
ericsilvagomez,

It's not really "repeating" the second sentence, because the second sentence is a general point about rent-control, whereas answer choice (D) is about specific municipalities. However, the general fact about rent control will definitely cause shortages of rental units in the kinds of municipalities talked about in the last sentence, so the statement in answer choice (D) Must Be True. It's true because of a combination of different parts of the stimulus, instead of just a repetition of one, but it's provably true all the same, which is all it needs to be to be the correct answer for a Must Be True question.

Robert Carroll

Get the most out of your LSAT Prep Plus subscription.

Analyze and track your performance with our Testing and Analytics Package.